Chứng minh tập hợp các lũy thừa của 2 trên bảng chữ cái ternary là không thường xuyên.


9

Nó đơn giản để thấy rằng quyền hạn của 2 trên bảng chữ cái {0,1} là thường xuyên vì 10 là một biểu hiện thường xuyên cho nó.

Nhưng sức mạnh của 2 đại diện trong ternary dường như là không thường xuyên. Bơm bổ đề hoặc các lớp dư lượng khó áp dụng vì dường như có rất ít mẫu trong số các chuỗi. Làm thế nào để tôi giải quyết nó?

Nói chung, với những quyền hạn nào của k đại diện trong cơ sở , tập hợp có thường xuyên không?r


1
Một quan sát dễ dàng là nếu cả k và r đều là lũy thừa của cùng một số nguyên thì tập hợp là chính quy. Tôi đoán rằng điều ngược lại cũng xảy ra, nhưng tôi không có bằng chứng.
Tsuyoshi Ito

1
Tôi nghĩ rằng đây là một bài tập trong cuốn sách của Sipser.
Zeyu

1
Hình như bài tập về nhà.
Warren Schudy

Câu trả lời:


12

Đây là một giải pháp thay thế (có giải thích chi tiết) bằng Định lý Myhill-Nerode. Tôi sẽ sử dụng cơ sở 2 để dễ đọc, nhưng bằng chứng tổng quát hóa cho các cơ sở tùy ý r , k không phải là lũy thừa của cùng một số nguyên.32r,k

(1) Hiển thị rằng với bất kỳ chuỗi ternary , tồn tại một chuỗi y khác sao cho x ylũy thừa của 2 .xyxy2

Chứng minh: Với bất kỳ nào (gọi n là số mà nó đại diện), kc { 0 , Hoài , 3 k - 1 } , tồn tại y sao cho x y đại diện cho 3 k n + c . Trong thực tế, điều này đặc trưng cho tất cả các số x y có thể đại diện. Do đó, việc tìm y tối thiểu sao cho x y có lũy thừa bằng 2 phụ thuộc vào việc tìm số nguyên k nhỏ nhấtxnkc{0,,3k1}yxy3kn+cxyyxy2ksao cho chúng ta có một số lũy thừa trong khoảng [ 3 k n , 3 k ( n + 1 ) - 1 ] . Lấy log cơ sở 2 , chúng ta cần tìm k sao cho chúng ta có một số nguyên trong khoảng [ k log 3 + log x , k log 3 + log ( x + 1 ) ] (bỏ - 12[3kn,3k(n+1)1]2k[klog3+logx,klog3+log(x+1)]1đây là iffy, nhưng đơn giản hóa các tính toán không dựa vào nó). Lưu ý rằng việc thay đổi chỉ ảnh hưởng đến phần 3 log k , vì vậy chúng ta có thể tìm thấy một k giúp chúng ta tùy ý gần với một số nguyên.kklog3k

(2) Cho một số và tối thiểu tương ứng với y , cho thấy rằng có tồn tại một chuỗi x' như vậy mà tối thiểu tương ứng y ' phải được lớn hơn y . Lặp lại điều này cho chúng ta vô số các lớp chuỗi tương đương.xyyy

Outline Proof: Vì , được cho một xyk tương ứng của nó , chúng ta luôn có thể tìm thấy một số x = 2 m x trong đó log ( 2 m x + 1 ) - log ( 2 m x ) đủ nhỏ để không có số nguyên nào trong [ k log 3 + m + log x ,log2mx=m+logxxykx=2mxlog(2mx+1)log(2mx) . Lưu ý rằng chúng tôi đang ngầm sử dụng thực tế là k log 3 + log x không bao giờ có thể là số nguyên.[klog3+m+logx,klog3+log(2mx+1)]klog3+logx


10

Nói chung, với những quyền hạn nào của đại diện trong cơ sở r , tập hợp có thường xuyên không?kr

Câu hỏi của bạn là một tập hợp con của Định lý Cobham (Alan Cobham, Về sự phụ thuộc cơ bản của các bộ số có thể nhận ra bởi automata hữu hạn, Lý thuyết về hệ thống điện toán 3 (2): 186--192, 1969, doi: 10.1007 / BF01746527 ):

Đặt là tập hợp các số nguyên không âm và cho mn là các số nguyên dương độc lập nhân. Sau đó S là dễ nhận biết bởi automata hữu hạn trong cả hai m -ary và n ký hiệu -ary khi và chỉ khi nó là cuối cùng kỳSmnSmn

Ở đây bằng cách nhân độc lập một nghĩa là không tồn tại khác không q sao cho m p = n q . Cobham trích dẫn Buchi đối với trường hợp cụ thể của bạn, quyền hạn của một số k trong cơ sở r , đó là nhận biết chỉ khi kr là multiplicatively phụ thuộc .pqmp=nqkrkr

Nếu bạn quan tâm đến kết quả này, có một cuộc khảo sát khá hay của Véronique Bruyère, Georges Hansel, Christian Michaux và Roger Villemaire ( Bộ số nguyên logic và nhận ra, Bản tin của Hiệp hội toán học Bỉ 1 (2), 1994, PDF ), cũng cho thấy mối quan hệ với số học Presburger.p


5

Bổ đề bơm ngụ ý rằng có một chuỗi như cho một số b , c , d sao cho mỗi x n là một lũy thừa của k . Vậy log k ( x n ) = d n log k ( r ) + c o n s t +xn=c+b(rdn1)/(rd1)b,c,dxnk luôn là một số nguyên, vì vậy log k ( r ) là hợp lý.logk(xn)=dnlogk(r)+const+o(1)logk(r)

Đây là một lời giải thích về công thức đó cho . Bổ đề bơm cho các chuỗi u , v , w sao cho mọi chuỗi x n = u v n w là lũy thừa của k . Giải thích các chuỗi này dưới dạng số và viết de cho độ dài của vw tương ứng, x n = u r d n + e + v r d ( n - 1 )xnu,v,wxn=uvnwkdevwlà một sức mạnh củad. Vậy x n - x n - 1 =(u r d -u+v) r d ( n - 1 ) + e . Viếtb=(u r d -uxn=urdn+e+vrd(n1)+e+vrd(n2)+e++vre+wdxnxn1=(urdu+v)rd(n1)+e c = x 0 - b ta có x n = c + b ( r d n - 1 ) / ( r d - 1 ) .b=(urdu+v)rec=x0bxn=c+b(rdn1)/(rd1)


(1) Tôi nghĩ rằng bạn cần một thuật ngữ như để đối phó với phần tiền tố của việc bơm. (2) Tôi không thể theo dõi lý do tại sao d n log k r + O ( 1 ) luôn luôn là một số nguyên ngụ ý log k r là hợp lý. Chính xác hơn, như được viết, nó là sai bởi vì bạn có thể tìm thấy 0 ε n < 1d n log k r + ε n là một số nguyên. Tôi nghĩ rằng bạn cần một cái gì đó cụ thể hơn ký hiệu O. erdndnlogkr+O(1)logkr0εn<1dnlogkr+εn
Tsuyoshi Ito

Nếu bạn viết thuật ngữ chung là nói, thì sự khác biệt của hai thuật ngữ có thể được xem là có dạng b r d n . xrdn+e+yrd(n1)+e+yrd(n2)+e++yre+zbrdn
Colin McQuillan

It is constant +o(1), that is not the same as O(1).
domotorp

@domotorp: I think that the item (2) in my previous comment was referring to a portion which was edited in the 5-minute window, but I am not sure. Maybe I could misread the answer.
Tsuyoshi Ito

@Colin: I am not sure what you are claiming by your last comment. It seems to me that your argument shows that a term like erdn is necessary.
Tsuyoshi Ito

4

Let L be the set of powers of 2 encoded in base 3. The encoding of 4n in base 3 ends with 1, whereas the encoding of 24n in base 3 ends with 2. Hence L=LΣ1 is the encoding of all powers of 4.

m>0log3(m+1)4n4n+1n>04nlog34n+1

Let N={log34n:n0} (this is a Beatty sequence). Suppose that L were regular. Then so would L be. This implies that the set of lengths of words in L is eventually periodic, and so N is eventually periodic. In particular, N would have rational asymptotic density. However, N has asymptotic density 1/log34, which is irrational.

Khi sử dụng trang web của chúng tôi, bạn xác nhận rằng bạn đã đọc và hiểu Chính sách cookieChính sách bảo mật của chúng tôi.
Licensed under cc by-sa 3.0 with attribution required.